LSAT and Law School Admissions Forum

Get expert LSAT preparation and law school admissions advice from PowerScore Test Preparation.

 Administrator
PowerScore Staff
  • PowerScore Staff
  • Posts: 8916
  • Joined: Feb 02, 2011
|
#41648
Complete Question Explanation
(The complete setup for this game can be found here: lsat/viewtopic.php?t=6962)

The correct answer choice is (A)

If T and V are the only side dishes selected, then the third side dish—W—is not selected. Without W, P cannot be selected (third rule). Additionally, N and V cannot both be selected (fifth rule), which means that N is not selected either. The “unassigned” group is thus completely determined, forcing the remaining three variables to all be selected:
PT65_D11 LG Explanations_game_#3_#16_diagram 1.png
The question stem asks us to determine a pair of variables, each of which must be selected. In other words, we are looking for two variables both of which must be selected. Since the local condition yields only one solution, validating the correct answer choice should not be terribly difficult.

Answer choice (A) is the correct answer choice, because F and G must both be selected, as shown in the local diagram above.

Answer choice (B) is incorrect, because N cannot be selected.

Answer choice (C) is incorrect, because P cannot be selected.

Answer choice (D) is incorrect, because N cannot be selected.

Answer choice (E) is incorrect, because P cannot be selected.

Get the most out of your LSAT Prep Plus subscription.

Analyze and track your performance with our Testing and Analytics Package.